ไม่สามารถเล่นวิดีโอนี้
ขออภัยในความไม่สะดวก

UNE QUESTION QUI REND FOU 🤯🤯🤯

แชร์
ฝัง
  • เผยแพร่เมื่อ 5 พ.ย. 2022
  • ⬇️ Lien vers la vidéo en question ⬇️
    • QUEL EST MON ÂGE ? 🤔
    Un type de question qui m'a longtemps rendu un peu fou tant on dispose de si peu d'éléments pour la résoudre..
    Voici l'énoncé.
    180 divisé par un nombre donne un reste de 2.
    Quel est le reste si on divise 1047 par ce nombre ?
    A) 0
    B) 1
    C) 2
    D) 3
    E) 4
    ⬇️⬇️ Tu veux la solution pour devenir solide en maths 💪 ? C'est par ici ⬇️⬇️
    ☞ hedacademy.fr

ความคิดเห็น • 249

  • @HououinKyoumaSG
    @HououinKyoumaSG ปีที่แล้ว +61

    J'ai utilise la division euclidienne des le départ, on sait que 180=ax+2
    x: le nombre qui divise
    a:un entier qu'on ne connait pas
    Et ax=178 et 2 le reste
    Ensuite j'ai fait la division de 1071 par 180
    1071=5×180+171
    D'où :
    1071=5(ax+2)+171=5ax+10+171
    1071=5ax+181
    Or 181=ax+2+1=ax+3
    Donc :
    1071=5ax+ax+3
    =6ax+3
    Donc le reste est 3

    • @Valerian-_-
      @Valerian-_- ปีที่แล้ว +1

      pareil

    • @dr-blood_9981
      @dr-blood_9981 ปีที่แล้ว +2

      J'ai fait à peu près pareil
      J'ai fait 180=nx+2
      Avec x=un nombre qu'on connaît pas
      Et n:un entier naturel qui divise
      nx+2=180
      nx=178
      J'ai décomposé 178 en facteurs premier
      :178=89×2
      Et j'ai testé 2 et 89
      2 ne marche pas car 180=2×90 +0
      89 marche car 180=2x89 +2
      J'ai ensuite divisé 1071/89
      Ca donne 1071=89×12 +3
      La réponse est donc 3

    • @feandil666
      @feandil666 ปีที่แล้ว +2

      j'ai fait pareil, je trouve ca bien plus simple, pas besoin de chercher quel est le diviseur

    • @Guill188
      @Guill188 ปีที่แล้ว +1

      la même méthode en précisant à la fin que le diviseur a n’était pas 1, 2 ou 3 puisque 180 divisé par 1 2 ou 3 ne donne pas de reste.

    • @Sashao99
      @Sashao99 ปีที่แล้ว

      J'ai fais pareil et je suis parti du principe que a = 2, puisque de toute façon la quantité ax est proportionnelle à x.

  • @austr1an
    @austr1an ปีที่แล้ว +10

    Avec les congruences :
    Soit x le diviseur inconnu,
    180 = 2 [x]
    178 = 0 [x]
    => 6*178 = 6*0 [x]
    1068 = 0 [x]
    1068+3 = 3 [x]
    1071 = 3 [x]
    Donc le reste de la division de 1071 par x est 3

    • @guillaumes-c7020
      @guillaumes-c7020 ปีที่แล้ว

      mais merci j'ai fait la même chose il se prend trop la tête a trouver le diviseur mdmmdrrr

    • @philipperoux8926
      @philipperoux8926 ปีที่แล้ว

      C est le plus simple et le plus efficace ..

    • @lapichfamily7595
      @lapichfamily7595 ปีที่แล้ว

      @@philipperoux8926 Oui mais c'est faux, encore faut-il montrer que 3

  • @themieljadida4459
    @themieljadida4459 ปีที่แล้ว +4

    180=2[n]
    Comme 1071=5×180+171
    Alors 1071=5*2+171[n]
    =181[n]
    =2+1[n] car 181=180+1
    C_a_d que le reste de 1071 par n est 3.
    Mais je ne me suis pas rendu fou !

    • @youssef5814
      @youssef5814 ปีที่แล้ว +1

      Bravo, c'est la meilleure réponse

  • @Playistful
    @Playistful ปีที่แล้ว +27

    Moi je suis parti du principe que 178 est un multiple du nombre inconnu donc n’importe quel multiple de 178 sera aussi un multiple de ce nombre, donc j’ai divisé 1071 par 178, j’ai trouvé 6 en quotient, hors 178 x 6 = 1068, il reste donc 3 pour arriver à 1071, d’où la réponse

    • @fredericfournier5662
      @fredericfournier5662 ปีที่แล้ว +1

      Super, c'est rapide...

    • @faridfredo3918
      @faridfredo3918 ปีที่แล้ว +1

      J'ai fait de même,c'est plus simple

    • @alestane2
      @alestane2 ปีที่แล้ว +4

      Il faut vérifier que le nombre inconnu est plus grand que le 3 trouvé pour affirmer que 3 est le reste. Sinon on sait juste que 1071 est congru à 3 modulo le nombre inconnu.
      Il faut donc vérifier que le nombre inconnu ne pouvait être ni 2 ni 3
      C'est immédiat parce que 180 est divisble par 2 et par 3, et on sait que divisé par le nombre inconnu il reste 2, mais ça pourrait être moins évident avec d'autres valeurs numériques.

    • @BlackSun3Tube
      @BlackSun3Tube ปีที่แล้ว

      @@alestane2 Ca ne peut pas être 2 d'entrée de jeu, puisque le reste de la première division est 2 :)
      On sait donc déjà que c'est un nombre supérieur à 2 à la lecture de l'énoncé.
      Ni 3 car 178 n'est pas divisible par 3 :)

  • @charlesbacany3186
    @charlesbacany3186 ปีที่แล้ว +6

    Avec les modulo ça marche tout seul
    180 =2 mod q
    Donc 900 =10 mod q et 171 = -7 mod q d’où par addition 1071= 3 mod q ..

    • @alestane2
      @alestane2 ปีที่แล้ว +1

      J'ai fait pareil (du coup j'efface ma réponse). Mais il faut encore justifier que q > 3 pour pouvoir affirmer que 3 est le reste de la division par q
      180 est divisible par 2 et par 3, et on sait que divisé par q il reste 2, donc q ne peut être ni 2 ni 3 (ni 1) donc q est supérieur à 3
      C'est évident ici mais ça pourrait l'être moins avec d'autres valeurs )numériques.

  • @armand4226
    @armand4226 ปีที่แล้ว +17

    Les mathématiques (bien expliquées) sont EXTRAORDINAIRES !!!!! 😍

    • @AngelinaJolie734
      @AngelinaJolie734 ปีที่แล้ว

      Mais quand c'est mal expliqué, ça devient vite fouillis. Ici, il suffisait de décomposer 178 en facteurs premiers, puis de prendre le plus petit des facteurs supérieurs à 2, donc 89. Après, on faisait simplement la division euclidienne de 1071 par 89.

    • @armand4226
      @armand4226 ปีที่แล้ว

      @@AngelinaJolie734 Et tu oses trouver ça plus simple ?
      Explique d'abord comme tu décomposes en nombre premier.
      C'est pas tout de le dire, faut aussi le démontrer.
      Ensui

    • @AngelinaJolie734
      @AngelinaJolie734 ปีที่แล้ว +1

      @@armand4226 Primo, les multiples de 2, 3 et 5 sont facilement identifiables, de même que ceux de 11 inférieurs à 1000. Avec ça, on peut déjà faire un gros débroussaillage.
      Secundo, on sait que 7*13=91, ce qui est juste au-dessus de 89, donc on peut sucrer ces deux-là. Après, pour 89, inutilse de chercher un autre nombre premier plus grand que 13, puisqu'il faudra forcément le multiplier par un nombre inférieur à 7, et on sait déja que 89 n'est pas divisible par 2 (donc également par 4 et 6), 3 ou 5. Et c'est donc un nombre premier.
      Tertio, pour ceux qui ont la fainéantise de faire ce genre de calcul, il existe des listes de nombres premiers très facilement trouvables sur Internet ou ailleurs.

    • @armand4226
      @armand4226 ปีที่แล้ว

      ​​​@@AngelinaJolie734 😅😅😅 Sacrée Angelina !!
      Ton raisonnement se tient pour les fortiches en maths, pas pour les bourrins comme moi.
      Tu crois que je sais de tête que 7x13 = 91 ?
      Sans machine ou sans écrire la multiplication, je ne le sais pas.
      😵‍💫
      Et puis des phrases comme " ... il faudra forcément multiplier par un nombre inférieur à 7 .. " sont totalement incompréhensibles pour moi, sans plus d'explications.

    • @AngelinaJolie734
      @AngelinaJolie734 ปีที่แล้ว +1

      @@armand4226 Là, c'est la logique même :
      Si on a a*b = c, si on augmente la valeur de a, il faudra forcément diminuer celle de b pour conserver l'égalité. ;)

  • @gusgus8501
    @gusgus8501 ปีที่แล้ว +5

    c'est génial ! par pitié, faites encore plus d'arithmétique 😀

  • @Minishoupiii
    @Minishoupiii 9 หลายเดือนก่อน

    Quelle belle et simple explication, tout montre la logique et la manière d'exposer le problème. Même à un âge avancé (59 ans), j'aime me creuser les méninges pour arriver à la solution d'un problème. Je calcule de tête quand je le peux, car je ne suis pas la génération "calculatrice" et ça fait du bien au cerveau.
    Merci encore pour tous ces raisonnements dans la bonne humeur.

  • @MsQuentin55
    @MsQuentin55 ปีที่แล้ว +3

    Je passe le Tage Mage dans 10 jours et c’est exactement le type de questions qui me fait peur ;) merci d’avoir passé du temps pour nous l’expliquer !

  • @smec63
    @smec63 ปีที่แล้ว +2

    J'adore ta manière d'expliquer. J'aurais voulu avoir un prof de maths comme toi !

  • @kazeman1698
    @kazeman1698 ปีที่แล้ว +1

    il y avait plus simple vu qu’il fallait trouver le reste et pas le nombre, j’ai juste appliqué les propriétés des congruences :
    si R est le reste de K alors n*R est le reste de n*K et R-m est le reste de K-m
    =>
    180 a pour reste 2,
    alors 6*180=1080 a pour reste 6*2=12
    donc 1071=1080-9 a pour reste 12-9=3

  • @erickbourdin4039
    @erickbourdin4039 ปีที่แล้ว +1

    Je suis passé par l égalité 180=dq+2 donc d=178/q
    1071=dp+r on remplace d 1071=178.(p/q) +r
    Donc r est le reste de la division de 1071 par 178 … soit 3
    Voilà voilà

  • @Anolyia
    @Anolyia ปีที่แล้ว +3

    Personnellement, j'ai juste utilisé les congruences. Vu que 180 est congru à 2 modulo 178, j'ai calculé que 1071 est congru à 3 modulo 178. Il resterait donc 2 réponses avec 0 ou 3. Vu que le reste de 180 par ce nombre est 2, alors ce nombre est strictement supérieur à 2. Par ailleurs, 178 n'est pas divisible par 3 donc la réponse est 3.

    • @touhami3472
      @touhami3472 ปีที่แล้ว

      Si tu utilises les congruences, pas besoin de chercher le diviseur n:
      180=2[n]
      Donc 1071=6×180-9
      =6×2-9=3[n]
      Conclusion: le reste de 1071 par n est obligatoirement 3 que n soit 89 ou un multiple de 89 inférieur à 180.

    • @Anolyia
      @Anolyia ปีที่แล้ว

      @@touhami3472 Si le diviseur est 3, alors le nombre serait congru à 0 modulo n. C'est pour ça qu'il fallait l'éliminer

    • @touhami3472
      @touhami3472 ปีที่แล้ว

      @@Anolyia n est tel que:
      180=2[n] donc n ne peut être : 2, 3,4,5,6,9,10,.... bref n ne peut être un diviseur de 180, c'est une condition de l'énoncé.

    • @ferdinandtrefle912
      @ferdinandtrefle912 ปีที่แล้ว

      t'as choisi 178 comme diviseur et t'as montré par congruences que le reste est 3 modulo 178 (= le diviseur) , cela suffit pour conclure, d'autant plus que ton justificatif à savoir que 3 ne divise pas 178 n'est pas correct

  • @rinkio9044
    @rinkio9044 ปีที่แล้ว

    Soit n le diviseur
    Il existe un k tel que 180 = k×n + 2
    Il existe un j et un p tels que 1071 = j×n + p (p

  • @bboynouri697
    @bboynouri697 ปีที่แล้ว

    Bravo et merci pour du partage des astuces et des différentes approches, ça aide quand on a pas un bon niveau ou perdu des concepts!

  • @philipperoux8926
    @philipperoux8926 ปีที่แล้ว

    Avec les congruences, c' est le plus simple , sachant que 1071 = 6x178 + 3.
    178 congru à 0 modulo n
    Donc 1071 congru à 3 modulo n.
    Les congruence ne sont hélas plus au programme de terminale..
    Comme les intégrales...

  • @ComliveJim69
    @ComliveJim69 7 หลายเดือนก่อน

    Ben j'ai cherché le produit de nombres premiers de (180-2=178) = 89x2, ça pouvait pas être 2, j'ai divisé 1071 par 89 j'ai trouvé 12x89+3.
    Il n'y a qu'une réponse possible que j'ai (3), je cherche même pas plus loin (diviser par 178).
    C'est pas une question qui m'a rendu fou. J'ai trouvé ça plutôt très simple par rapport a plein d'autres exercices :D

  • @762x51mm
    @762x51mm ปีที่แล้ว +2

    Résolution en passant par les degrés d'angles.
    ---------------------------------------------------------------------------
    1071 c'est proche de 1080.
    180 c'est la moitié de 360.
    1080 c'est 360 * 3. Les multiples de 360 c'est comme les puissances de 2 c'est important de les connaitre (360°= 1 tour) (1080° = 3 tours)
    Du coup 1071 c'est 1080 - 9 (moins neuf IMPORTANT POUR LA SUITE).
    Et le rapport de 1080 sur 360 c'est 6 (IMPORTANT POUR LA SUITE).
    Donc si je divise 1080 par ce nombre, je dois obtenir un reste de 12 (même si le nombre est plus petit que 12 et que dans ce ca là la division n'est pas terminée, mais c'est pas grave).
    Il me reste 12 quand je travaille avec 1080. Or on doit travailler avec 1071, soit 9 de moins. Je retire 9 de 12 et j'obtiens 3.

    • @michelbernard9092
      @michelbernard9092 ปีที่แล้ว

      Après cette solution ésotérique, je vais m'empresser de consulter mon astrologue qui utilise une table circulaire et des cartes.

  • @marcelrenou4141
    @marcelrenou4141 ปีที่แล้ว +1

    Je suis parti de 180=qn+2
    1080 = 180x6 donc 1071 = 180x6 -9 = (qn+2)x6 -9
    donc 1071 = 6qn + 12 - 9 = 6qn +3
    180 étant multiple de 1, 2, et 3, le diviseur n est nécessairement supérieur strictement à 3 et donc 3 est plus petit que mon diviseur et donc nécessairement mon reste (il y a unicité du quotient et du reste d'une division euclidienne)

    • @hediarfaoui5255
      @hediarfaoui5255 ปีที่แล้ว

      180 = kn +2
      178 = kn
      178 ×6 = 6kn
      1068 = 6kn
      1068 +3 = 6kn +3
      1071 = pn +3.

  • @Sashao99
    @Sashao99 ปีที่แล้ว +3

    En voyant le sujet, je me suis dis : encore une question bien tordue, quelle est la technique qui va résoudre ça? Finalement 2 minutes après je l'avais résolu... Je vais plutôt aller me remettre sur les tableaux de variation parce que ça fais 20 ans que je n'en ai pas refais. A l'époque je n'en saisissais pas l’intérêt alors qu'aujourd'hui je remarque bien que ça permet de résoudre des problèmes de modélisation.

  • @z-ryk
    @z-ryk 8 หลายเดือนก่อน

    180 = xy+2
    => 178 divisible par x et 180 pas divisible par x
    => x = 89 ou 178, car 178 n'est divisible que par (1,2,89 et 178) et 1 et 2 sont des diviseurs de 180
    1071 = xz + r
    => 1071 - r est divisible par x
    Si on divise 1071 par 89 :
    * 1071 = 89*12 + 3
    Si on divise 1071 par 178 :
    * 1071 = 178*6 + 3
    Dans les 2 cas le reste est 3

  • @claudeBgf
    @claudeBgf ปีที่แล้ว +1

    Vu qu'on annonce qu'il y a une solution au problème, on prend 178 qui donne obligatoirement un reste de 2.
    On voit rapidement que 1071/178 donne environ 6 (178*5 = 890, si on ajoute 180 ça donne 1070).
    On voit déjà ici que le reste c'est 3 (180-178 + 1), sinon suffit de constater que 178 * 6 va se terminer par 8, donc le reste est forcément 3

    • @Frank-kx4hc
      @Frank-kx4hc ปีที่แล้ว +1

      Oui, mais le 'ou' ici (=qcm), n'est pas forcément exclusif .

    • @claudeBgf
      @claudeBgf ปีที่แล้ว +1

      Mon opinion est que la question est suffisamment claire pour en tirer cette conclusion, la question étant posée pour avoir une solution, sinon elle n'a pas vraiment de sens.
      Sinon, il suffit de poser l'équation, ça prend juste un peu plus de temps: Et le temps, c'est souvent ce qui manque dans ce genre d'exercice.

  • @palicot
    @palicot ปีที่แล้ว

    Exercice intéressant, mais il existe une preuve beaucoup plud immédiate...
    On notant n le diviseur, et r le reste on peut poser:
    180=n*q+2 et 1071=n*p+r
    Il s'agit de voir que
    1071 = 180*6-9 = (n*q+2)*6-9=n*6q+3=n*p+r
    Puis par unicité de la division euclidienne, on identifie p=6q et r = 3.
    Pour compléter proprement la preuve, il faut juste montrer que n est strictement supérieur à 3...
    Par l'absurde, si n = 1 ou 2 ou 3, alors le reste de la division euclidienne de 180 par n serait 0 dans les trois cas et donc l'hypothèse de l'énoncé serait fausse. Donc n > 3.
    Cqfd.

  • @doncamilo6143
    @doncamilo6143 ปีที่แล้ว

    Congruences (spé Maths S). Soit n le nombre, ~ dénote la congruence, [n] modulo n : 180 ~ 2 [n] -> 180*6 ~12 [n] -> 180*6-9 ~ 3 [n] -> 1071 ~ 3 [n]. Réponse : 3. CQFD.

  • @DavidStrainchamps
    @DavidStrainchamps ปีที่แล้ว

    180=aq+2. a=178/q
    1071/a=q*1071/178
    a entier différent de 1
    178=2*89
    Donc soit a=2 soit a=89
    Donc reste =1 ou reste = 3

  • @alainamar898
    @alainamar898 ปีที่แล้ว +2

    Pourquoi faire long quand on peut faire court:?
    4 lignes :
    180 = xa+2,
    1071=6X180-9,
    1071= 6(xa+2) - 9 ,
    1071= 6xa +12 -9 = 6xa+3

    • @gerardb8044
      @gerardb8044 ปีที่แล้ว

      j'ai fait pareil et c'est bien plus simple

  • @ipatchymakouli415
    @ipatchymakouli415 ปีที่แล้ว

    Plutot facile, 180/2=90 donc c'est celui juste avant (89) qu'on va prendre pour qu'il reste 2.
    Un peu au pif vu que pas doué en calcul mental, j'utilise 90 plutot que 89 pour me rapprocher de 1071. 90*10=900, et 90*2=180, la Somme des deux c'est donc 90*12=1080, on est au plus près de 1071.
    Ensuite on soustrait 12, vu que c'est pas 90 qu'on utilise mais 89, donc 89*12=1068. Et 1071-1068=3, voilà mon raisonnement.

  • @touhami3472
    @touhami3472 ปีที่แล้ว

    Pas besoin de chercher le divisur.
    1071=5×180+171 donc le reste par n est 5×2+171=180+1 soit r=2+1=3 car 180 par n est 2.

  • @valentinprovost4978
    @valentinprovost4978 ปีที่แล้ว +1

    Comme d’habitude : TROP BIEN !!!!

  • @Kelgha06
    @Kelgha06 ปีที่แล้ว

    Pour moi, la division par 178 n'est pas nécessaire.
    On sait que 1071 = 89x2n + r ou 1071 = 89x(2n+1) + r
    Et comme 178 = 89x2 on a alors 1071 = 89x2n + r = 178n + r ou 1071 = 89x(2n+1) + r = 178n + (89 + r)
    Or on sait grâce aux réponses proposées que le reste est strictement inférieur à 89. Il suffit donc de faire la division de 1071 par le plus petit diviseur possible, soit 89.

  • @fredericfournier5662
    @fredericfournier5662 ปีที่แล้ว

    Voilà comment je m'y suis pris.
    J'ai posé en ligne les deux divisions euclidiennes.
    r est le reste à calculer, on a les deux quotients q1 et q2 et le diviseur d qui est le même pour les deux lignes.
    180 = d x q1 + 2
    1071 = d x q2 + r
    Avec d comme terme identique, on parvient à l'égalité suivante :
    r = 1071- 178(q2/q1)
    Rien ne peut sortir de ça pour l'instant...
    On reprend les deux lignes en négligeant les restes:
    180 = d x q1
    1071 = d x q2
    On trouve que q2/q1est très proche de 6. (1071/180 = 5,95)
    Pour finir on reprend la ligne
    r = 1071- 178(q2/q1)
    On trouve r = 3 (en prenant q2/q1 = 6 et pas q2/q1 = 5,95 ce résultat étant une approximation au départ puisqu'il vient d'un calcul ne tenant pas compte des restes, restes dont on ne connaît d'ailleurs pas l'un des deux...)
    Pffffh bon, un peu long mon truc.
    Je vais me pencher sur l'explication de Kazeman c'est tellement plus joli... 😄

  • @BOLNRenovation
    @BOLNRenovation ปีที่แล้ว

    Excellent!!!!

  • @alina-reyes
    @alina-reyes ปีที่แล้ว

    Trouvé en moins de 15 secondes, et j'ai du mal à m'intéresser à une longue explication. Je dis ça parce que je me demande pourquoi on doit tellement compliquer les choses pour trouver un résultat qui requiert juste un rapide calcul mental. N'y aurait-il pas un moyen d'apprendre les maths autrement ? Je le demande naïvement car il y a très longtemps que je n'ai pas fait de maths, mes années de lycée sont à des décennies d'aujourd'hui.

  • @adam4478
    @adam4478 ปีที่แล้ว +1

    180 = a*n + 2 donc on a bien 178 = a*n
    1071/178 ≈ 6 donc on s'interresse à 6*178
    6*178 = 1068
    6*178 = 6*a*n = b*n
    on a donc 1068 = b*n
    d'où 1071 = b*n + 3

  • @yahyabensouda3075
    @yahyabensouda3075 ปีที่แล้ว

    « Tu fais qui tu veux avec qui tu veux ». Meilleure définition de l’associativité de la multiplication. Apparemment la multiplication est échangiste.

  • @victorinbrunel
    @victorinbrunel ปีที่แล้ว +1

    J'ai fait les congruences :
    1. Division Euclidienne de 1071 par 180 : 1071 = 5 * 180 + 171
    2. Soit x le reste cherché et m le "nombre", on a 1071 ≡ x [m]
    Donc 5 * 180 + 171 ≡ x [m]
    5 * 2 + 171 ≡ x [m] (on remplace 180 par 2)
    10 + 171 ≡ x [m]
    181 ≡ x [m]
    1 + 180 ≡ x [m]
    1 + 2 ≡ x [m]
    x ≡ 3 [m]
    Soit 1071 ≡ x ≡ 3 [m]
    3. 3 est le reste de la DE de 1071 par m ssi m > 3 :
    - m > 2 car le reste de 18O/m est 2
    - si m = 3, 180 / 3 = 60, soit un reste = 0 et non pas 2, donc m != 3
    Donc m > 3
    Le reste de la DE de 1071 par m est donc 3.

    • @capotthomas9719
      @capotthomas9719 ปีที่แล้ว

      c'est compliqué tout ça fallait pas se prendre la tête. L'énoncé nous dit 180 ≡ 2 (m). Donc 178 ≡ 0 (m) 178 ≡ 178x6 ≡ 1068 ≡ 0 (m). on ajoute 3 de part et d'autre. 1071 ≡ 3 (m)

  • @mathi04yt72
    @mathi04yt72 ปีที่แล้ว +2

    Perso j’ai tout fait de tête sur base de la miniature.
    J’ai d’abord trouvé que le nombre en question devait être un diviseur de 178 par logique. Ensuite j’ai pas cherché de diviseurs autre que 178. Perso j’ai de la facilité à multiplier par 2 de tête. Donc j’ai fait 178 + 178 = 356. Puis 356 + 356 = 712. Enfin 712 + 356 = 1068. 1071 - 1068 = 3 . Voilà pour moi

  • @jlpreda8559
    @jlpreda8559 ปีที่แล้ว

    On utilise la division euclidienne
    a=bq+r r étant plus petit que b
    si a=180 et r=2 alors bq=180-2=178
    On divise 1071 par le même nombre ici 178 reste plus qu’à faire la division euclidienne de 1071 par 178 donc 1071=178*6+3
    Donc le reste c’est 3

  • @rickydlayaute5387
    @rickydlayaute5387 ปีที่แล้ว

    Whaaa j'ai trouvé mais j'ai pas fait l'fier !! Merci Heda !!!👍😎

  • @mariama8716
    @mariama8716 ปีที่แล้ว +1

    Je fais ça en maths merci beaucoup❤❤

  • @cofbmaitres1177
    @cofbmaitres1177 ปีที่แล้ว +1

    J'ai directement fait la division euclidienne de 1071 par 178, sans même chercher les autres diviseurs de 178, car ça ne change rien au reste (puisque si un nombre divise 178, alors il divise aussi n'importe quel multiple de 178)

    • @kingminato5219
      @kingminato5219 ปีที่แล้ว

      Excellent

    • @jsuiquimoi
      @jsuiquimoi ปีที่แล้ว +1

      C'est aussi ce que je me suis dit, mais pour être tout à fait juste je crois il aurait fallu vérifier si 178 était divisible par 3 (ce que je n'ai pas fait mais qui est très facile/rapide), parce que si c'était le cas, le reste aurait aussi pu être 0. Il y aurait 2 solutions. Me trompé-je ?

    • @bazounet32
      @bazounet32 ปีที่แล้ว

      @@jsuiquimoi Parfaitement juste. Je serais aussi tomber dans le panneau, si le diviseur avait été 3, j'avais fait exactement aussi le même raisonnement.

    • @Phlifre78
      @Phlifre78 ปีที่แล้ว

      Sauf qu'avec 1160 (1071 + 89) ce raisonnement ne marche pas. Et pourtant la réponse serait la même.

    • @cofbmaitres1177
      @cofbmaitres1177 ปีที่แล้ว

      @@Phlifre78 J'ai simplement dit que "si un nombre divise 178, alors il divise aussi n'importe quel multiple de 178", mais je n'ai jamais dit que la réciproque était vraie, et heureusement parce qu'elle est fausse, comme le prouve l'exemple de 1160. J'avoue que je n'avais pas pensé à vérifier la réciproque, mais on peut dire quand même qu'il fallait bien de toute façon que les deux restes soient égaux, puisqu'on ne peut cocher qu'une seule réponse

  • @raphaelharism6839
    @raphaelharism6839 ปีที่แล้ว

    J'ai fait 180*6=1080
    Donc 9 de moins que 1071
    Comme on a fait *6
    Donc le reste 2*6 =12
    12-9 = 3
    Le reste est 3
    En 2sec
    J'étais pas sur du tout de moi mais ca a marché
    Après j'ai trouvé pourquoi ca a marché cque j'ai fait. Mais parfois l'instinc est cool

  • @Vital11Aude
    @Vital11Aude ปีที่แล้ว

    autre méthode : j'ai noté le diviseur b et le quotient q donc 180=bq+2 donc q=(180-2)/b=178/b
    puis pour la seconde division, j'ai noté le diviseur b' et le reste r donc 1071=b'q+r=b'/b*178+r or 1071=178*6+3 donc r=3

  • @capotthomas9719
    @capotthomas9719 ปีที่แล้ว

    Les congruences dans ce genre d'exercice ça sauve la vie... 180 ≡ 2 (x) donc 178 ≡ 0 (x) delà, 178 ≡ 6x178 ≡ 1068 (x) et donc 1971 ≡ 3 (x). en quelques équivalences on tombe tout de suite sur le résultat

  • @eyamarime649
    @eyamarime649 ปีที่แล้ว +1

    Génial 👌👌

  • @gaetanbouthors
    @gaetanbouthors ปีที่แล้ว

    soit x le nombre en question, alors 178=kx avec k un entier. Alors 1071=178*6+3 soit (6k)x+3 donc 1071 divisé par x donne 6k reste 3. (le seul cas ou ca ne marcherait pas c'est x inferieur a 3 mais on voit bien que c'est absurde car on obtiendrait pas 2 comme reste en divisant 180 par 1, 2 ou 3) pour une question qui rends fou, elle se resout en quelques secondes

  • @raycharlz4937
    @raycharlz4937 ปีที่แล้ว

    Pour une fois, je l'ai trouvée tellement facile à trouver instinctivement que j'ai cru que c'était encore un piège, mais non.
    Pour avoir un reste égal à 2, il suffit de prendre le chiffre divisé par 2 et de retrancher 1 : 180/2 - 1 = 89. 180/89 donne un reste de 2.
    89 x 12 = 1068 (ça ne prend même pas une minute à calculer).
    Donc le reste est 3.

    • @stephmar2634
      @stephmar2634 ปีที่แล้ว

      je suppose que tu parles de 89, et non de 79?

    • @raycharlz4937
      @raycharlz4937 ปีที่แล้ว

      @@stephmar2634 tout à fait, j'ai modifié du coup.

  • @sea34101
    @sea34101 ปีที่แล้ว

    180 = p.q +2
    On multiplie par 6
    6×180 = 1080 = 6.p.q + 12
    Donc 1071 = 6.p.q + 3
    Le reste c'est 3
    Pour être honnête, ma première solution fut de faire comme vous, mais je n'étais pas satisfait car cela consiste à deviner le diviseur et il y a toujours la possibilité d'en oublier un.
    Ici, comme le reste c'est deux, le diviseur est plus grand que trois. Donc on sait que reste c'est trois.

  • @V3Yoshi
    @V3Yoshi ปีที่แล้ว

    J'ai juste fait 180 / 178 = 1 x 178 + 2, donc j'ai fait 1 071 / 178 = 178 x 6 + X = 1 068 + X, donc X = 1071 - 1068 = 3

  • @malicksanoh2643
    @malicksanoh2643 ปีที่แล้ว

    180=ax+2
    J’ai pris ax=178
    178/2=89
    Puis 1071/89= 12,03 et donc 1072/12 aura pour reste 3☺️

  • @michelrobert9085
    @michelrobert9085 ปีที่แล้ว

    Excellent

  • @Sebmagic2
    @Sebmagic2 ปีที่แล้ว

    On regarde combien de fois 180 permet de s'approcher de 1071.
    180*6=1080, donc le reste de la division de 1080 par ce nombre est 2*6=12.
    Donc pour 1071=1080-9, le reste est 12-9=3.

  • @AArrakis
    @AArrakis ปีที่แล้ว

    Le plus simple, c’est 180-2=178. On décompose en nombres entiers. 178 = 2*89. Ensuite on teste 1071, 1070, 1069 etc divisés par 178 ou 89 jusqu’à trouver celui qui donne unreste egal à 0. Or 1068 = 178*6=89*12. D’où la réponse est trois.

    • @italixgaming915
      @italixgaming915 ปีที่แล้ว

      Tu t'embêtes encore moins en retranchant 890 à ton nombre (c'est un multiple de 89 et de 178, nos DEUX diviseurs candidats - il en a oublié un dans sa démo), et tu tombes sur 181. Et là comme tu es paresseux, tu réutilise l'hypothèse du début ^^

    • @touhami3472
      @touhami3472 ปีที่แล้ว +1

      On s'en fout des candidats.
      Le plus simple est de traduire les conditions du texte:
      180=2[n] : cad 2 est le reste 180 par n,
      1071=6×180-9
      = 6×2-9[n] car le reste 180 par n est 2
      =3[n] cad le reste de 1071 par n qu'on connait pas encore, est 3.
      Si on nous demande, en deuxième question, de déterminer n: on utilise les résultats précédents et on en déduit n:89 ou 178.

  • @mahikannakiham2477
    @mahikannakiham2477 ปีที่แล้ว

    Mon truc simple, on sait que 180 modulo 89 = 2, donc en multipliant par 6 on trouve que 1080 modulo 89 = 12, et puis en soustrayant 9 on trouve que 1071 modulo 89 = 3

  • @jige1225
    @jige1225 ปีที่แล้ว

    5:20 Svp pour une prochaine fois, écrivez la division euclidienne plutôt que faire un calcul par essais et erreurs, les étudiants ont besoin de méthode !

  • @Ambryne1
    @Ambryne1 ปีที่แล้ว

    Vraiment chouette mais tu parles un tout petit peu trop vite 😉👌

  • @orenaud2109
    @orenaud2109 ปีที่แล้ว +1

    Bonjour
    La division par 178 ne sert à rien la décomposition en facteurs premiers 1 2 et 89 le reste est 2 donc la division par 89 suffit
    Et on trouve bien un reste de 3 avec 12 fois 89

    • @italixgaming915
      @italixgaming915 ปีที่แล้ว

      Si, elle sert, parce que le nombre avec lequel on divise peut être 89 ou 178. Les deux marchent. Il en a oublié un dans sa démo.

    • @RaoulVanDePetrucci
      @RaoulVanDePetrucci ปีที่แล้ว

      J'ai failli commettre la même erreur concernant 178. M'est revenu à l'esprit que 2 est le reste, erreur évitée.
      180 / 178 , le reste est 2.

  • @marchamelin66
    @marchamelin66 ปีที่แล้ว

    De tête. Si pour 180 il reste 2 donc 180*6=1080 ce qui fait 9 de trop. Comme j'avais 6*2 a enlever, il me reste 12-9=3. Le reste est 3 sans connaître le diviseur

  • @Kapeutini
    @Kapeutini ปีที่แล้ว

    Wow, merci tu me fais aimer les maths

  • @philippegibault6889
    @philippegibault6889 ปีที่แล้ว

    Je n'ai pas encore vu la vidéo, mais comme Sans Nom, je pense que la réponse est d) soit 3.
    Pour rappel, la division euclidienne est si on a a/b, alors a = b.q + r où q est le quotient et r le reste.
    Soit x le nombre par lequel on divise.
    Diviser par 180, ça veut dire que l'on a (1) 180 = x.q + 2.
    Diviser par 1071, ça veut dire que l'on a 1071 = x.q' + r. x, q, q' sont entiers, bien évidement.
    On a donc le système suivant:
    180 = x.q + 2
    1071 = x.q' + r. x
    Là, en appliquant le principe de mon professeur de DEA en Physique, on est dans le caca. Car selon ce principe, si on a n inconnues, il faut n équations pour déterminer une solution.
    Ici, on n'a seulement que 2 équations pour 3 inconnus. Mon professeur dirait que l'on a un problème mal posé.
    C'est pour ça, par exemple, que les équations de Maxwell sont, par exemple, un problème bien posé. On a E, B, p(densité de charge) et j et on a justement 4 équations...
    On va donc ici travailler la ligne 1.
    Ce qui donne x = 178/q.
    On injecte ça dans la ligne 2.
    Soit 1071 = 178Q + r avec Q = q'/q. On remarque d'emblée que Q doit être un entier, car r (ce que l'on cherche) est aussi un entier.
    On a r = 1071- 178Q. Comme r >=0, on vérifie que Q

  • @davidbla7846
    @davidbla7846 ปีที่แล้ว +1

    les math donnent vraiment la joie

  • @michelbernard9092
    @michelbernard9092 ปีที่แล้ว +1

    Je profite de cet exo pour vous donner un truc facile pour trouver le reste d'une division euclidienne avec une calculette : au résultat, de la division qui est affiché par la calculette, vous enlevez la partie entière, et ensuite vous multipliez le résultat par le diviseur.
    Exemple sur la calculette 1071/178= 6,01685... vous enlevez 6 ça donne 0.0168853 , vous multipliez par 178 et ça donne 3

    • @Frank-kx4hc
      @Frank-kx4hc ปีที่แล้ว

      Ou directement : 1071-6×178= reste de 1071 par 178.
      C'est la règle :
      dividende = qoutient *diviseur +reste.
      Sinon : il y a une touche qui donne le reste . Mais attention :
      18 touche" a+b/c" 14 affiche 1,2/7
      Ce qui signifie : 1=q .toujours vrai ,
      Mzis 2 /7 doit 2/7=4/14 ==> le reste de 18/14 est 4.

    • @michelbernard9092
      @michelbernard9092 ปีที่แล้ว

      @@Frank-kx4hc Oui, mais le "6" n'est pas si évident de tête ! alors qu'enlever le nombre entier "6" de 6.0168.. c'est facile.

    • @Frank-kx4hc
      @Frank-kx4hc ปีที่แล้ว

      @@michelbernard9092 mais le "6" dont je parle est la partie entière de ton résultat 6,016... donc "6" est le quotient de 1081par 178 .
      Donc on connaît : le dividende, le diviseur et le quotient.
      D'où le reste est 1071-6*178.

  • @winstons.8303
    @winstons.8303 ปีที่แล้ว

    Continue frero tu explique super bien ??

  • @alainqueyras372
    @alainqueyras372 ปีที่แล้ว

    Trop fort !!!👍

  • @Timpourlesintimes
    @Timpourlesintimes ปีที่แล้ว

    Salut ! J'ai un problème que j'ai vu il y a qq années et je n'arrive pas à poser l'équation pour le résoudre. Si qq'un a la méthode :
    Tous les jours je fais le même trajet à la même vitesse et je mets le même temps. Un jours je décide de rouler 20km/h plus vite et je gagné 2 minutes. Le lendemain je roule 20 km/h moins vite et je perds 3 minutes. Quelle est la distance parcourue ?

    • @charlesbacany3186
      @charlesbacany3186 ปีที่แล้ว +1

      Les réponses : v=100km/h , t= 12min, d=20km
      Tu traduis chaque phrase par une equation en appelant d la distance, v la vitesse et t le temps exprimés respectivement en km, km/h et h
      1ere equation d=vt
      2eme equation d=(v+20)(t-2/60)
      3eme equation d= (v-20)(t+2/60)
      C’est un système de 3 equations à 3 inconnues qui se résout facilement..

    • @charlesbacany3186
      @charlesbacany3186 ปีที่แล้ว

      La 3e equation dans la 2e parenthèse c’est t+3/60..

    • @Timpourlesintimes
      @Timpourlesintimes ปีที่แล้ว

      Merci

  • @Fexghadi
    @Fexghadi ปีที่แล้ว +1

    Au pire même si on avait eu 1160, le reste de la division par 178 aurait été de 92, et donc ne donnait pas une réponse valide. Pas bien grave si les deux diviseurs possibles ne donnent pas la même bonne réponse :-)

    • @alestane2
      @alestane2 ปีที่แล้ว

      Oui, comme c'est un QCM et que la réponse "indéterminé" n'est pas proposée, on peut prendre un raccourci.

  • @michellepivert2490
    @michellepivert2490 ปีที่แล้ว

    Je ne regarde pas encore la solution mais je me demande si une fois trouvé le nombre on est autorisé à faire la division de 1071 par ce nombre pour donner le reste ou alors faut-il faire une autre démonstration ? donc je vais donner le nombre ; 180 est divisible par 2 mais 180 n'est pas divisible par 2x , il reste 2 donc 180 = 2x + 2 soit 90 = x + 1 x = 89 bon maintenant je continue quand même sans regarder 1071 est divisible par 3 89 ne l'est pas donc si je divise par 89 il va rester 3 ! j'ai pas eu besoin de voir .

    • @michellepivert2490
      @michellepivert2490 ปีที่แล้ว

      Heureusement que je n(ai pas regardé , j'aurais fini par ne plus rien comprendre ! Je sais cela ressemble à un coup de poker mais j'ai décomposé 1071 en produits de facteurs
      1071 = 3^2 X 7 X 17 le reste peut-il être supérieur à 3 1071 = 89y + 4 1071 est divisible par 3 4 ou 2 ne le seront jamais . Donc 1071 - 3 = 1068 = 3 X 4 X 89
      3 fois 4 fois 89 , bizarre vous avez dit bizzare !

  • @morphilou
    @morphilou ปีที่แล้ว

    Lol g mis moins de 20 sec avec 1 raisonnement bcp plus simple ==> pour 180 il reste 2 dc pour 360 il reste 4 et pour 720 il reste 8 et pour 1080 il reste 12 dc reste 3 pour 1071
    Après je suis à l aise en calcul mental dc g vu direct le 108p mais même un mec lampda mettra pas 1mn pour résoudre cela

  • @arezkisaadouni3439
    @arezkisaadouni3439 ปีที่แล้ว

    Merci

  • @nicks8106
    @nicks8106 ปีที่แล้ว

    J'ai posé 178/x = y et 2/x = 2/178y
    En résolvant ce système, l'on trouve bien x=178 et donc

  • @Jean575575
    @Jean575575 ปีที่แล้ว

    178 = 89x2 reste 0.
    1071 = 890 + 181 = (890+178)+3
    = (89x12)+3 reste 3

  • @maths_plus7092
    @maths_plus7092 ปีที่แล้ว

    merci

  • @mickaelb.3931
    @mickaelb.3931 ปีที่แล้ว

    Whaaaa.... difficile, celle-là !

  • @claudinehembise7078
    @claudinehembise7078 ปีที่แล้ว

    Génial pour les « nuls » comme moi. Merci

  • @note10litegalaxy76
    @note10litegalaxy76 ปีที่แล้ว

    On cherche résultat dans N alors si cherchait résultat dans R ou bien dans C

  • @pepemouss100
    @pepemouss100 ปีที่แล้ว

    Vous êtes forts moi je n’y comprends rien....je vais faire un cap de boulanger....

  • @baba796
    @baba796 ปีที่แล้ว

    Bientôt une application du lemme des restes chinois ?

  • @cyruschang1904
    @cyruschang1904 ปีที่แล้ว

    180 ÷ N = M + 2/N => 180 = NM + 2
    1071 ÷ 180 = 6 (180) - 9 = 6 (NM + 2) - 9 = 6 NM + 12 - 9 = 6 NM + 3
    La réponse D) 3

  • @cronos351
    @cronos351 ปีที่แล้ว

    180-2)/2=89
    1071/89=12. ...
    12*89=1068
    1071-1068=3

  • @mqgjqrgj
    @mqgjqrgj ปีที่แล้ว

    Mathématiquement, il faut prendre en considération les chiffres 1 et 2 qui sont également des solutions. Du coup la solution est À, B ou D et non justement D.
    Bref.

  • @youssef5666
    @youssef5666 ปีที่แล้ว

    meme reflexion sauf que pour le choix je me suis dis soit le meme reste soit des restes qui ne sont pas dans les reponses donc a eliminer pour trouver le bon

  • @anotherguy7395
    @anotherguy7395 ปีที่แล้ว +4

    En vrai je comprends pas pourquoi la question est dur, j'ai juste fait un logique simple :
    Si le reste c'est 2 c'est forcément que la réponse est supérieur a 2 donc 0/1/2 sont impossible
    Et le reste d'une division euclidienne par 4 est forcément impaire si la valeur divisé est impaire donc 4 est éliminé

    • @pandaroux9465
      @pandaroux9465 ปีที่แล้ว +2

      Joli ... Sauf que le postulat de base est sans fondement. Diviser 7 par 3 donne pour reste 1, qui est inférieur à 3. Donc l'exclusion est sans fondement

    • @yugapillon1343
      @yugapillon1343 ปีที่แล้ว +1

      Nan, ça marche pas
      La réponse qu'on cherche, c'est le reste
      Le diviseur peut effectivement pas être 0, 1 ou 2, ou 4, mais comme ce qu'on cherche, c'est pas les diviseurs, on peut pas les éliminer

  • @gigou97
    @gigou97 ปีที่แล้ว

    Pour mon bla réponse est de 3 et le nombre en question 89 mais je laisse l'expliquer 🥳

  • @kirkpatrick415
    @kirkpatrick415 ปีที่แล้ว

    Je ne comprends pas bien le départ car on sait d'office que 0, 1, 2 et 4 (4x45 = 180) ne marchent pas donc il n'y a que 3 qui correspond à la 1ere question

  • @jacquesperio3017
    @jacquesperio3017 ปีที่แล้ว

    Allez au fait!

  • @mandokelteir2376
    @mandokelteir2376 ปีที่แล้ว

    De mon côté
    175 x 6 = 1050
    6 x 3 = 18
    1050 + 18 = 1068
    Donc il reste 3.
    Sachant que 178 n'est pas divisible par 3, et qu'il restait déjà 2 la dernière fois, pas besoin de chercher plus loin.

    • @armand4226
      @armand4226 ปีที่แล้ว

      Je n'ai rien compris ...
      Tu es trop fort. Comment fais-tu pour avoir l'idée de partir de 175 ?
      Et surtout ensuite, qu'est-ce qui t'a amené l'idée de multiplié 6 par 3 ?
      D'ou sort ce 3 ? Pourquoi pas 4, 2 ... ?
      Je suis sur le cul ...
      Je repose bien ma question : comment as-tu ces idées ????

    • @mandokelteir2376
      @mandokelteir2376 ปีที่แล้ว

      @@armand4226
      J'ai voulu multiplier 178.
      Mais le plus facile c'est de passer par les multiples de 25, qui sont simples à multiplier.
      175 x 2 = 350.
      350 x 3 = 1050.
      On a donc fait 175 x 2 x 3 donc 175 x 6.
      Il nous reste 3 x 6 à faire (car on a multiplié 175 et pas 178).
      Donc 3 x 6 = 18
      On ajoute tout ce qu'on a fait
      18 + 1050.
      1068.
      On est à 3 unités de 1071, le résultat souhaité.
      C'est donc forcément la solution, comme je l'ai dit précedemment.
      J'ai tout simplement voulu me simplifier la tâche en commençant par des multiples faciles, ce qui m'a directement amené au résultat final, comme souvent dans ce genre de problème.

    • @michelbernard9092
      @michelbernard9092 ปีที่แล้ว

      Bon, là il suffisait juste de trouver un nombre entier N tel que 180=N+2.. pas trop compliqué quand même !

    • @armand4226
      @armand4226 ปีที่แล้ว

      @@michelbernard9092 Pas pour tout le monde, pas pour tout le monde.

  • @SCF_Empire
    @SCF_Empire ปีที่แล้ว +1

    🎉🎉🎉🎉👌👌👌👌👌

  • @Easyldur_
    @Easyldur_ ปีที่แล้ว

    On part de
    180 = yx+2
    1071 = zx + ?
    Donc on peut faire
    1071 = (180 * 6) - 9
    1071 = 6yx + 12 - 9
    1071 = 6yx + 3

  • @ait-melloulitihad4607
    @ait-melloulitihad4607 ปีที่แล้ว

    T'es trop fort!!!

  • @mouradbelkas598
    @mouradbelkas598 ปีที่แล้ว

    le reste est 3.. 180/178, 1071 / 178 = 6 reste 3

  • @lazare93
    @lazare93 ปีที่แล้ว

    Facile, réponse D), il faut dire que √7921 est un nombre premier lol.

  • @jeanmasse7935
    @jeanmasse7935 ปีที่แล้ว

    en dix secondes j’ai découvert la solution ,méthode de calcul ( 1948 ) .

  • @aolivier8262
    @aolivier8262 ปีที่แล้ว

    Bah sur une question comme ca c est pas de la technique, c est du calcul mental pur et dur. Il m a fallu moins d une minute temps de lecture inclus

  • @alainreseau6777
    @alainreseau6777 ปีที่แล้ว

    180 = xy + 2
    1071 = 180*6 -9 = 6xy + 12-9 = 6xy +3 donc le reste est 3

  • @jeanmasse7935
    @jeanmasse7935 ปีที่แล้ว

    avec la méthode de 1948 en primaire léa solution est découverte en moins de 10 seconde

  • @nicolanicoca3505
    @nicolanicoca3505 ปีที่แล้ว

    la question qui rend fou la question qui tue... PLS....

  • @charognard37
    @charognard37 ปีที่แล้ว

    Alors facteurs premiers de 178 (180-2) :
    2x89
    La division par 2 ne pouvant pas donner un reste de 2 c'est donc 89 le diviseur.
    on fait donc 1071 MOD 79 = 3
    ou 1071 - INT(1071/89)*89 = 3 pour ceux que MOD révulse

    • @italixgaming915
      @italixgaming915 ปีที่แล้ว +1

      Tu as fait la même faute que lui, il faut aussi considérer 178.

    • @charognard37
      @charognard37 ปีที่แล้ว

      @@italixgaming915 Ah oui merde c'est vrai ;)

  • @sirene18
    @sirene18 ปีที่แล้ว

    Ça m'a plu :-)

  • @Photoss73
    @Photoss73 ปีที่แล้ว

    Vu les différents commentaires, tous les chemins mènent à Troyes. 🙂

  • @jonathanmax7502
    @jonathanmax7502 ปีที่แล้ว

    2=180-ax
    2 = 180 -178
    Donc ce 178 est forcément un multiple de ce nombre (x). Or les nombres qui divisent 178 sans reste sont 1,2,89 et bien sûr 178.
    Mon principal candidat est donc 89.
    2=180-2×89
    On peut diviser alors 1071 par 89. Ça va pour 12 fois et le reste est 3. Résolu 👍😜

    • @jonathanmax7502
      @jonathanmax7502 ปีที่แล้ว

      Sans regarder la vidéo 😜

    • @alestane2
      @alestane2 ปีที่แล้ว

      @@jonathanmax7502 Vous avez juste traité le cas x=89 en ignorant la possibilité x=178. Ca ne marche que parce que l'énoncé et les réponses du QCM impliquent qu'il y a une solution, sinon la réponse est juste mais le raisonnement est incomplet.
      Essayez avec 1160 au lieu de 1071; la bonne réponse est "on ne peut pas dire lequel entre 3 et 92", alors que votre raisonnement donnerait juste 3.

    • @jonathanmax7502
      @jonathanmax7502 ปีที่แล้ว

      @@alestane2 Tu as vu juste, j'ai ignoré le cas de 178 même si cela donne encore le même résultat (3). Je te remercieerci pour la remarque 👍

    • @italixgaming915
      @italixgaming915 ปีที่แล้ว

      Il ne faut pas faire comme lui et écarter 178. 178 marche aussi.

    • @touhami3472
      @touhami3472 ปีที่แล้ว

      @@alestane2 soit n entier tel 180=2[n]
      Alors 1160=6*180+80
      =92[n]
      Mais attention : ce 92 n'est pas forcément le reste dans la division euclidienne mais un reste modulo n.
      On peut montrer que le reste r de la D.E de 1160 par n est tel que : 2r=6[n] ce qui donne bien r=3 .
      Conclusion : il existe bien un entier n tel qui vérifie les DEUX CONDITIONS de l'énoncé: 180=2[n] ET 1160= r[n] où r dans {0,1,2,3,4}
      Maintenant, on peut chercher ce n qui vérifie ces deux conditions : n=89.